Monica is rollerblading along a trail in Tropical Park. She stopped for water after rollerblading for 2.45 miles. She continued on the trail for another 3.04 miles, then stopped for lunch. She made her final stop after rollerblading for another 2.8 miles. How many miles did Monica rollerblade?

Answers

Answer 1

Answer:

8.29

Step-by-step explanation:

If you add them all together your answer should be 8.29

Answer 2

Answer:

8.29

Step-by-step explanation:

hope it helps! :))


Related Questions

An animal shelter has $2500 in its reserve fund. The shelter charges $40 per animal placement and would like to have at least $4000 in its reserve fund. Write an inequality to represent this situation.

Answers

Answer:

Step 1

Write and solve the inequality:

2,500 + 40a ≥ 4,000, or 40a ≥ 1,500

a ≥ 37.5

Step 2

If the shelter places 30 cats and 10 dogs,

or 40 animals, that will be enough to meet

its goal, because a = 40 is a solution to the

inequality a ≥ 37.5.

Help me please ?!!!!

Answers

Answer:

The slope of the line is -1

Step-by-step explanation:

Slope of a Line

Suppose we know a given line passes through points A(x1,y1) and B(x2,y2). The slope can be calculated as follows:

[tex]\displaystyle m=\frac{y_2-y_1}{x_2-x_1}[/tex]

From the table in the image, we select the points (1,4) (3,2). The slope is:

[tex]\displaystyle m=\frac{2-4}{3-1}=\frac{-2}{2}=-1[/tex]

Now select two other points like (5,0) (7,-2)

[tex]\displaystyle m=\frac{-2-0}{7-5}=\frac{-2}{2}=-1[/tex]

Following the same procedure with any other pair of points, we'll get the very same result:

The slope of the line is -1

this is the picture that goes with my question.​

Answers

Answer:

first u should make your equation: y= -1/5 (x-5) - 3

y= -1/5x +1 -3

y= -1/5 x -2

then u should setting some number like 1 ,2 ..... instead of *x* and when u done this u should dissolve it (now your *y* is came out too) so you have your x,y now

According to the points you get, you put it on the chart and connect it

if you do not understand something, tell me to draw on paper so you can understand better

Which function corresponds to the table?
A) y = 3x - 2
B) y = 2x + 3
C) y = -2x + 3
D) y = -3x + 2

Answers

Answer:

c is the correct answer

{(-5, 4), (-4,-1), (-2, 1), (0, 4), (1,3)}
i need to find the domain and range

Answers

domain : (-5, -4, -2, 0, 1)
range : (4,-1, 1,4,3)

The required domain is {-5, -4, -2, 0, 1} and range is {4, -4, 1, 4, 3}.

As given in the question, for the ordered pairs given {(-5, 4), (-4,-1), (-2, 1), (0, 4), (1,3)} domain and range of the system is to be determined.

What is the range?

Range, it is the set of the values that come out to an outcome for a certain mathematical operation.

What is simplification?

The process in mathematics to operate and interpret the function to make the function or expression simple or more understandable is called simplifying and the process is called simplification.

Here,
Given the ordered pair are,
{(-5, 4), (-4,-1), (-2, 1), (0, 4), (1,3)}

Values in the place of abscissa x and ordinate y are the value of a set of domains and ranges respectively.
So domin =  {-5, -4, -2, 0, 1}
and range =  {4, -4, 1, 4, 3}

Thus, the required domain is {-5, -4, -2, 0, 1} and range is {4, -4, 1, 4, 3}.

learn more about the range here:
https://brainly.com/question/12239390

#SPJ2


Can you please help meeeeeeeee

Solve for x.

x2+6x+6=0

Enter your answers in the boxes.

x = or x =

Answers

Answer:

so x= -3+ root three

and x = -3- root three

You have to use the quadratic formula to solve

Please answer this question... please
Select the graph that represents the equation y= 2/3x-3

Answers

It’s the second one. Pick the 2nd one
ITS THE LAST ONE!!! NOT THE SECOND ONE

Marty’s dog can run 3 miles in 20 minutes. What is the dog’s speed in miles per hour?
How do i get the answer?

Answers

Since there is 60 minutes in a hour you would multiply 20*3 to get the hour. Now since you multiplied the minutes by 3 you also multiply the amount of miles giving you 9 miles per hour

Is the statement true or false? If false, give a counterexample.

For all real numbers a, b and c, a(b + c) = ab + bc.

A. true
B. false; a(b + c) = ab − ac
C. false; if a = b = c = 1, then 1(1 + 1) ≠ 1(1) + 1(1)
D. false; if a = 1, b = 2, and c = 3, then 1(2 + 3) ≠ 1(2) + 2(3)

Answers

Answer:

D. false; if a = 1, b = 2, and c = 3, then 1(2 + 3) ≠ 1(2) + 2(3)

Step-by-step explanation:

A is false, because the a is being distributed/ being multiplied to all terms inside the parenthesis, and not the term b.

B is also false. There is no indicated negative signs.

C is also false because 1(1 + 1) is EQUAL to 1(1) + 1(1)

D is true.

If 52 men can do a piece of work in 35 days in how many days will 28 men to it?

Answers

28 mando the peace of work in 41 days

by divida 52 and 35 and multiiply with28

expand (2x+3) (2x-5)​

Answers

Answer:

4x^2−4x−15

Split using the sum and difference of the formula, the simplify

4x^2+16x+15

the up arrow means power of 2

so first number is 4x to the powder of 2

simplify
12+ 1233-987

Answers

Answer:

258

Step-by-step explanation:

Answer:

258

Step-by-step explanation:

Follow the order of PEMDAS

complete each proof​

Answers

Answer:

a) given

b) subtraction property

d) 3x+8 -8= 2-8

e) 3x= -6

f) division property

g) x=-2

Step-by-step explanation:

Solve the inequality from part A for x. Then use the drawing tools to graph the solution set on the number line. Solving lenear equations

Answers

Answer:

On the number line, mark the endpoint at 10 with a closed circle.  

Next, draw a ray to the right of 10. ➡️

➡️

Step-by-step explanation:

solve the inequality for x:

1,558 + 60x ≥ 2,158

60x ≥ 600

x ≥ 10

On the number line, mark the endpoint at 10 with a closed circle.  

Next, draw a ray to the right of 10. ➡️

What is inequality?

An inequality is a relation which makes a non-equal comparison between two numbers or mathematical expressions.

here, we have,

solve the inequality for x:

1,558 + 60x ≥ 2,158

60x ≥ 600

x ≥ 10

To learn more on inequality click:

brainly.com/question/24853349

#SPJ2

Evaluate x/y when x=9/4 and y=3/5

Answers

Answer:

[tex]\displaystyle x/y=\frac{15}{4}[/tex]

Step-by-step explanation:

Division of fractions

To calculate the division of fractions a/b and c/d, it's usually easier to multiply a/b by the reciprocal of the denominator, that is:

[tex]\displaystyle \frac{\frac{a}{b}}{\frac{c}{d}}=\frac{a}{b}\cdot \frac{d}{c}[/tex]

We will evaluate x/y when x=9/4 and y=3/5:

[tex]\displaystyle \frac{\frac{9}{4}}{\frac{3}{5}}=\frac{9}{4}\cdot \frac{5}{3}[/tex]

[tex]\displaystyle=\frac{45}{12}[/tex]

Simplifying:

[tex]\displaystyle \boxed{x/y=\frac{15}{4}}[/tex]

Mrs. Bartz was going shopping for Halloween Candy for her first grade class. If
a bag of 25 Skittle's costs $4.00, how much would one bag of Skittles cost?

Answers

Answer:

The answer is 0.16

Step-by-step explanation:

because if you add .16 25 times or if you see the other answers it cant be 6.25 or 1.05 you think it could be 0.82 but its not

Answer:

0.16

Step-by-step explanation:

A group of 80 ants went to collect some honey. 1/2 lost their way and 1/4 got eaten by a passing anteater. How many ants came back?

Answers

Answer:

23

Step-by-step explanation:

Matthew is going to drive from Salt Lake City, Utah, to Las Vegas, Nevada. His car travels a consistent

number of miles

per gallon of gasoline and he has some gasoline in his tank already. Matthew writes the

equation 25(x + 3) = 424, where x represents the number of gallons of gasoline he will need to purchase to

complete the trip.

Identify the meaning of each of the constants in Matthew's equation in the context of the problem, then solve

the equation to determine how many gallons of gasoline he must purchase.

Answers

Answer: 14 gallons of gasoline.

Step-by-step explanation:

given data:

25(x + 3) = 424

where x is the number of gallon of gasoline needed to purchase.

Solution.

25(x + 3) = 424

first we open the bracket

25x + 75 = 424

collect like terms

25x = 424 – 75

25x = 349

divide both sides by 25

25x/25 = 349/25

x = 13.96

x = 14

mathew needs to get 14 gallons of gasoline.

WILL MARK BRAINLIEST

What is the range of the function f(x)=1/3x+1, when the domain is {-1,1,4}

a {0, 2, 4}

b {-1, 1, 3}

c {2/3, 4/3, 2}

d {-2/3, 5/3, 4}

Answers

Answer:

D.

Step-by-step explanation:

SORRY IF THIS IS WRONG IM NOT GOOD IN MATH BUT I TRY

​If the area of a square poster is 625 square inches. Josh needs to it to fit into a frame that has dimensions of 12 inches. How many inches does he need to cut off each dimension?

Answers

Answer:

13 inches

Step-by-step explanation:

To solve this problem we need to figure out the length of each side. Caculate the length of the square poster by finding out the square root. 25 times 25= 625. We know that th dimentions of the poster is 25. 25-12= 13

Pls help hurry I need to find the slope

Answers

Answer:

a. -1

Hope this helps!

The answer would be C.

CAN SOMEONE PLEASE HELP ME ASAP PLEASEE ANYBODY LITERALLY ANYONE OUT THERE PLEASE. Determine m∠ABQ and determine m∠BCR.

Answers

Answer:

ABQ  = 139

BCR is the same as QBC since they are alternate interior angles

BCR = 41

Step-by-step explanation:

CBQ and SCB are same side interior angles so they add to 180

2a-9  + 5a +14 = 180

Combine like terms

7a +5 = 180

Subtract 5 from each side

7a = 175

Divide by 7

7a/7 = 175/7

a = 25

ABQ  is the same as SCB  since they are corresponding angles so

ABQ = SCB = 5a+14 = 5*25+14 = 125+14 = 139

BCR is the same as QBC since they are alternate interior angles

BCR = QBC = 2a-9 = 2*25 -9 = 50-9 = 41

Which shows the scale factor used to dilate triangle XYZ to form triangle X'Y'Z'?
A( 1/2 x, 1/2 y)
B (1/3 x, 1/3 y)

Answers

The answer you would be looking for is A. (1/2 x, 1/2 y)

Answer:

B

Step-by-step explanation:

A is wrong lol, trust me.

A carpenter has less than 120 minutes to spend painting furniture each day. Today, he has spent 30.5 minutes painting a desk. Now he will paint x chairs, each of which takes 12.5 minutes. What is the maximum number of chairs the carpenter can paint?

Answers

Answer:

Maximum number of chairs is 7.

Step-by-step explanation:

We are told he has less than 120 minutes to spend painting furniture each day.

Now he has spent 30.5 minutes painting a desk.

Also, he wants to paint "x" number of chairs with each chair taking 12.5 minutes.

Thus, we have;

30.5 + 12.5x < 120

Subtract 30.5 from both sides to get;

12.5x < 120 - 30.5

12.5x < 89.5

x < 89.5/12.5

x < 7.16

Thus, maximum number of chairs he can paint is 7

combine like terms to simplify the expression

Answers

1/2 K - 3/5 is the answer. You’re welcome and Brainliest would be appreciated :)

Answer:

[tex]\frac{1}{2} k-\frac{3}{5}[/tex]

Step-by-step explanation:

Step 1: Define expression

[tex]\frac{2}{5} k-\frac{3}{5} +\frac{1}{10} k[/tex]

Step 2: Change denominators to like terms

[tex]\frac{4}{10} k-\frac{3}{5} +\frac{1}{10} k[/tex]

Step 3: Combine like terms

[tex]\frac{5}{10} k-\frac{3}{5}[/tex]

Step 4: Simplify

[tex]\frac{1}{2} k-\frac{3}{5}[/tex]

I need help please.
M a t h

Answers

Answer:

option 2

Step-by-step explanation:

Answer:

Step-by-step explanation:

Here are the conditions.

The car must go at least 55 mph. That means that 55 is included. Use ≥

The car's speed must be greater than 55. That means s>55 Notice the open end of the arrow is towards the s. The point is towards 55. The large end must be towards the largest entity (s in this case).

The answer uses ≥

The answer is s ≥  55

does 3(6x - 2) - 7 = 18x - 13?​

Answers

Answer:

yes

Step-by-step explanation:

distribute 3(6x-2) to get 18x-6-7= 18x-13

combine -6 and -7 to get 18x - 13 = 18x - 13

hope this helps :))

Which of the following forms a proportion?
5/9 = 14/18 3/5 = 21/35 6/12 = 18/34 1/8 = 2/24

Answers

Answer:

3/5 = 21/35

Step-by-step explanation:

if you reduce 21/35 it is 3/5 so it is the same thing

Find m2 KLM ifm/NLM = 950
and mKIN = 46°.

Answers

Answer:

46

Step-by-step explanation:

Arron can join a gym for $19.99 per month plus an annual fee of $12.80 or he can pay $21.59 per month. He thinks the second option is better because he plans to use the gym for 10 months. Is Aaron correct ? Set up an equation (with variables on both sides) to describe the situation.

Answers

Answer: Aaron is wrong. The first option is better as it costs less than the second option.

Step-by-step explanation:

Arron can join a gym for $19.99 per month plus an annual fee of $12.80.

Since Aaron wants to spend 10 months at the gym, this will cost him:

= $12.80 + ($19.99 × 10)

= $12.80 + $199.9

= $212.7

Paying $21.59 for 10 months will be:

= $21.59 × 10

= $215.9

The first option is better as it costs less than the second option.

Other Questions
Where does Washingtons loyalty reside during the onset of tho conflict Which one ? A. B. C. or D? A wagon with a mass of 35 kg is pulled so that it accelerates from 0 m/s to 30 m/s in 15seconds.Remember to show all work!a. What is the acceleration of the wagon? b. What is the force required to get this acceleration? Plz help me What state of matter has a defined volume but undefined shape?A ) gasesB ) liquids and gasesC ) liquidsD ) solids and liquids 21. What does LCSY stand for? Can someone check to see if I got this right?!?? A bill can pass in the U.S. senate with a three-fifths vote. If there are 100 senators, how many are needed to pass a bill? where did martial arts originate? Can somebody Awnser these for me please I need help with this and can you explain how to do it?3(2y + 3) + y Describe 3 ways in which the federal government restricted immigration. What groups of people were most negatively affected by these laws? Answer the following questions using what you've learned from this unit. Write your answers in thespace provided. Be sure to show all work.1. Solve the equation.5(x - 4) + 3x - 9x = 6 - (2x + 5) + 8xStep 1: Use the distributive property to remove the parentheses. (2 points)Step 2: Collect any like terms on both sides of the equation. (2 points)Step 3: Bring all variable terms to one side of the equation. (Hint: Add x to both sides.) (2 points) Copy and complete:__:11= 1/3 :6 gimme a riddle lol like a hard one Which equation is equivalent to y=2/3x-6A. 2x+3y=-6B.3x-2y=6C.3x-2y=12D.2x-3y=18 You are running a 10 mile race. You run the first 3 miles in 24.7 minutes. Your goal is to finish the race in less than 1 hour and 20 minutes. What should you average running time (in minutes per mile) be for the remaining miles? The posture of the company is to produce high quality products with minimal impact on the environment. A. The country's posture was very defeated after losing many soldiers in the Vietnam War. B. The athlete forced himself to have a sad posture when he left his hometown's team. C. The ballet dancer's graceful posture was due to years of training his body. D. The movie star's posture to the media is that she is generous and wholesome. Use the commutative property of addition to rewrite and simplify the expression.10 + 6 In a sequence of numbers, a3=0, a4=4, a5=8, a6=12, and a7=16. Based on this information, which equation can be used to find the nth term in the sequence, an?an=4n12A sub n is equal to 4 n minus 12an=12n+4A sub n is equal to negative 12 n plus 4an=4n+12A sub n is equal to negative 4 n plus 12,an=12n4 Why is there no Save option in Word Online? Documents are automatically saved to the hard drive. Documents are automatically saved to OneDrive. Documents are automatically saved to RAM. Documents are automatically saved to a USB flash drive.